Question

In: Finance

Consider the three stocks in the following table. Pt represents price at time t, and Qt...

Consider the three stocks in the following table. Pt represents price at time t, and Qt represents shares outstanding at time t.

           Po      Q0        P1        Q1

A         87      100       92      100

B        47      200     42       200

C        94      200      104    200

What is the rate of return on the equal-weighted index of the three stocks?

O 1.92%

O 10.64%

O. 4.07%

O 4.39%

Solutions

Expert Solution

The Weightage of the Three stocks is Equal

So, Weights of each stock = 1/3

Total Value of Portfolio At Time 0

= Price of Stock A  * Quantity of Stock A * Weight of stock A + Price of Stock B * Quantity of Stock B * Weight of stock B + Price of Stock C * Quantity of Stock C * Weight of stock C

= 87*100*(1/3) + 47*200* (1/3) + 94*200*(1/3)

= $12,300

Now, The Total Value of Portfolio At Time 1

= Price of Stock A  * Quantity of Stock A * Weight of stock A + Price of Stock B * Quantity of Stock B * Weight of stock B + Price of Stock C * Quantity of Stock C * Weight of stock C

= 92*100*(1/3) + 42*200*(1/3) + 104*200*(1/3)

= $12,800

The Rate of return

= (Total Value of Portfolio At Time 1 - Total Value of Portfolio At Time 0) / Total Value of Portfolio At Time 0

= (12800-12300) / 12300

= 500/12300

= 4.065%

Or 4.07% (rounded off to nearest digit)

Option C is the correct answer


Related Solutions

Consider the three stocks in the following table.  Pt represents price at time t, and Qt represents...
Consider the three stocks in the following table.  Pt represents price at time t, and Qt represents shares outstanding at time t. Stock C splits two-for-one in the last period. Can you show work with steps and answer c please                         P 0       Q 0     P 1      Q 1     P 2      Q 2             A         80        100      85        100      90        100             B         60        200      45        200      40        200             C         110      200      120      200      60        400 c. Calculate the rate of return of the price-weighted index for the second period (t=1 to t=2).
Consider the three stocks in the following table. Pt represents price at time t, and Qt...
Consider the three stocks in the following table. Pt represents price at time t, and Qt represents shares outstanding at time t. Stock C splits two for one in the last period. P0 Q0 P1 Q1 P2 Q2 A 90 100 95 100 95 100 B 50 200 45 200 45 200 C 100 200 110 200 55 400 a. Calculate the rate of return on a price-weighted index of the three stocks for the first period ( t 5...
Consider the three stocks in the following table. Pt represents price at time t, and Qt...
Consider the three stocks in the following table. Pt represents price at time t, and Qt represents shares outstanding at time t. Stock C splits two-for-one in the last period. P0 Q0 P1 Q1 P2 Q2 A 100 100 105 100 105 100 B 60 200 55 200 55 200 C 120 200 130 200 65 400 Calculate the first-period rates of return on the following indexes of the three stocks: (Do not round intermediate calculations. Round your answers to...
Consider the three stocks in the following table. Pt represents price at time t, and Qt...
Consider the three stocks in the following table. Pt represents price at time t, and Qt represents shares outstanding at time t. Stock C splits two-for-one in the last period. P0 Q0 P1 Q1 P2 Q2 A 80 100 85 100 85 100 B 40 200 35 200 35 200 C 80 200 90 200 45 400 Calculate the first-period rates of return on the following indexes of the three stocks: (Do not round intermediate calculations. Round your answers to...
Consider the three stocks in the following table. Pt represents price at time t, and Qt...
Consider the three stocks in the following table. Pt represents price at time t, and Qt represents shares outstanding at time t. Stock C splits two-for-one in the last period. P0 Q0 P1 Q1 P2 Q2 A 85 100 90 100 90 100 B 45 200 40 200 40 200 C 90 200 100 200 50 400 a. Calculate the rate of return on a price-weighted index of the three stocks for the first period (t = 0 to t...
Consider the three stocks in the following table. Pt represents price at time t, and Qt...
Consider the three stocks in the following table. Pt represents price at time t, and Qt represents shares outstanding at time t. Stock C splits two-for-one in the last period. P0 Q0 P1 Q1 P2 Q2 A 84 100 89 100 89 100 B 44 200 39 200 39 200 C 88 200 98 200 49 400 a. Calculate the rate of return on a price-weighted index of the three stocks for the first period (t = 0 to t...
Consider the three stocks in the following table. Pt represents price at time t, and Qt...
Consider the three stocks in the following table. Pt represents price at time t, and Qt represents shares outstanding at time t. Stock C splits two-for-one in the last period. P0 Q0 P1 Q1 P2 Q2 A 90 100 95 100 100 100 B 50 200 45 200 35 200 C 100 200 110 200 55 400 What will the index be in day 2? (Hint: stock split happens in the evening. Do not round intermediate calculations. Round your answer...
Consider the three stocks in the following table. Pt represents price at time t, and Qt...
Consider the three stocks in the following table. Pt represents price at time t, and Qt represents shares outstanding at time t. Stock C splits two-for-one in the last period. P0 Q0 P1 Q1 P2 Q2   A 83       100       88       100       88       100         B 43       200       38       200       38       200         C 86       200       96       200       48       400       a. Calculate the rate of...
Consider the three stocks in the following table. P(t) represents price at time t, and Q...
Consider the three stocks in the following table. P(t) represents price at time t, and Q (t) represents share outstanding at time t. Stock C splits two for one in the last period. P(0) Q(0) P(1) Q(1) P(2) Q(2) A 90 100 95 100 95 100 B 50 200 45 200 45 200 C 100 200 110 200 55 400 First, calculate the price weighted indexes at t=0 and t=1. Based on the two numbers calculate the rate of return....
Consider the three stocks in the following table. Ptrepresents price at time t, and Qtrepresents shares...
Consider the three stocks in the following table. Ptrepresents price at time t, and Qtrepresents shares outstanding at time t. There are three time points 0, 1, and 2. P0 Q0 P1 Q1 P2 Q2 A 90 100 95 100 100 100 B 50 200 45 200 45 200 C 100 200 110 200 105 200 a) Form an equally weighted portfolio in stocks A, B, C at time 0 and hold it to time 1. Calculate the portfolio return...
ADVERTISEMENT
ADVERTISEMENT
ADVERTISEMENT